2
$\begingroup$

I'm quite new to differential forms/interior derivatives so if anyone could help with understanding the intuition behind proving this certain property step by step would be great.

Let $\iota_X$ denote the interior derivative of differential forms with a vector field $X$. Show that the operator: $$L_X:= \iota_X \ \circ d \ + \ d \ \circ \ \iota_X$$

satisfies $$L_X(\omega_1 \ \wedge \ \omega_2)=(L_X \omega_1) \ \wedge \omega_2 \ + \ \omega_1 \ \wedge \ (L_X \omega_2)$$

So far I have gotten to the expression:

$$\iota_X \ \circ \ [(d\omega_1) \ \wedge \ \omega_2 \ + \ (-1)^k\omega_1 \ \wedge \ d(\omega_2)] \ + d \ \circ \ [(\iota_X\omega_1) \ \wedge \ \omega_2 \ + \ (-1)^k\omega_1 \ \wedge \iota_X(\omega_2)]$$

by using the following properties:

$$\text{(1)} \ \ \ d(\omega_1 \wedge \omega_2)=d\omega_1 \wedge \omega_2 +(-1)^k(\omega_1 \wedge d\omega_2)$$

$$\text{(2)} \ \ \ \iota_X(\omega_1 \wedge \omega_2) = \iota_X(\omega_1) \wedge \omega_2 + (-1)^k\omega_1 \wedge \iota_X(\omega_2)$$

Edit: after substituting the properties again for a second time I am left with:

$$\iota_x \ \circ \ d\omega_1 \ \wedge \ \omega_2 \ + \ (-1)^k d\omega_1 \ \wedge \ \iota_x(\omega_2) \ + \ (-1)^k \iota_x \omega_1 \ \wedge d\omega_2 \ + \ (-1)^{2k} \omega_1 \ \wedge \ (\iota_X \ \circ \ d\omega_2) \ + \ d \ \circ \ \iota_X \omega_1 \ \wedge \ \omega_2 \ + \ (-1)^k(\iota_X \omega_1 \ \wedge \ d\omega_2) \ + \ (-1)^k(d\omega_1) \ \wedge \ \iota_X \omega_2 \ + \ (-1)^{2k}\omega_1 \ \wedge \ (d \ \circ \ \iota_x\omega_2)$$

$\endgroup$
16
  • 2
    $\begingroup$ Have you written down the right-hand side? How do the terms compare? $\endgroup$ Commented Feb 18, 2022 at 19:29
  • 2
    $\begingroup$ What do you mean “get rid of”? What you’ve written down so far expands to 8 terms. Do they simplify/combine to give the 4 terms on the right-hand side? Write it all out explicitly. $\endgroup$ Commented Feb 18, 2022 at 19:43
  • 2
    $\begingroup$ Your two formulas below show you how each type of term turns into two! $\endgroup$ Commented Feb 18, 2022 at 19:49
  • 2
    $\begingroup$ No, you already used property (2) to get this. But you need to compute $d(\dots)$ here, so use property (1). In the first term, you need to use (2) again, right? $\endgroup$ Commented Feb 18, 2022 at 22:32
  • 4
    $\begingroup$ Good job, you two $\endgroup$ Commented Feb 19, 2022 at 1:31

1 Answer 1

4
$\begingroup$

Following from above:

\begin{align} L_X(\omega_1 \wedge \omega_2) &= (\iota_X \circ d + d \circ \iota_X)(\omega_1 \wedge \omega_2) = (\iota_X \circ d)(\omega_1 \wedge \omega_2)+ (d \circ \iota_X)(\omega_1 \wedge \omega_2)\\ &= \iota_X \circ [(d\omega_1) \wedge \omega_2 + (-1)^k\omega_1 \wedge d(\omega_2)]+ d\circ [(\iota_X \omega_1) \wedge \omega_2+(-1)^k\omega_1 \wedge\iota_X(\omega_2)]\\ &= \iota_X \circ d\omega_1 \wedge \omega_2 + (-1)^{k+1} d\omega_1 \wedge \iota_X\omega_2 + (-1)^k \iota_X \omega_1 \wedge d\omega_2 + (-1)^{2k} \omega_1 \wedge \iota_X \circ d\omega_2 \\ & \qquad + d\circ\iota_X \omega_1 \wedge \omega_2 + (-1)^{k-1}\iota_X \omega_1 \wedge d\omega_2 + (-1)^k d\omega_1 \wedge \iota_X \omega_2 + (-1)^{2k}\omega_1 \wedge d\circ\iota_X\omega_2\\ &= (\iota_X \circ d\omega_1 \wedge \omega_2 + d\circ \iota_X\omega_1 \wedge \omega_2) + (\omega_1 \wedge \iota_X \circ d\omega_2 + \omega_1 \wedge d\circ \iota_X \omega_2)\\ &= (L_X\omega_1) \wedge \omega_2 + \omega_1 \wedge (L_X \omega_2)\\ \end{align}

Thank you for the help Ted!:)

$\endgroup$

You must log in to answer this question.

Start asking to get answers

Find the answer to your question by asking.

Ask question

Explore related questions

See similar questions with these tags.